« first day (2395 days earlier)      last day (2627 days later) » 

11:13 AM
@user8469759 I believe so. Alternatively, you can visualize it as a surface: f(x, y) = |(x, y)|
 
user84215
11:44 AM
Hello
 
user84215
can we have an uncountable set of real numbers that none of its element is limit point ?
 
No, intersect with [-n,n] and use Bolzano-Weierstrass
 
user84215
ok by bolzano it has limit point. But is it necessary this limit point belong to set ?
 
wow nice pictures .. where are you coding it?
 
@aminliverpool ah, I misread, the answer is still yes, but I don't remember a proof of the top of my head
 
user84215
12:00 PM
from where do you answer "yes" ?
 
It's a known fact
Every uncountable subset of R contains one of its limit points, you can probably find it on mse
 
@DHMO You there?
 
user84215
@AlessandroCodenotti I need an uncountable set of real numbers that none of its element is limit point. So by your statement, it never occurs.
 
$$\lim_{x->0} \frac{1}{\ln(1+x)}-\frac{1}{\ln(x+\sqrt(1+x^2)}$$ @DHMO Try finding this limit!
 
12:35 PM
Probably overkill, but I'd substitute $x=\sinh t$ so that the second term simplifies
 
12:49 PM
Is any of you familiar with splines functions? and related properties
(BSplines)
 
@AkivaWeinberger I see you and Zach talked about some interesting problem above. Want to restate?
@MateenUlhaq It's the "right" notion of a metric on a smooth surface/manifold.
The dot product on each tangent space gives you the ability to define arclengths of paths on the smooth surface.
That's why it's also known as a Riemannian metric.
 
Guys, any1 aware of video lecture regarding calculus of variations, any university lectures??
 
@BalarkaSen The complement of finitely many arcs (injective images of $[0,1]$) in $\Bbb R^2$ is path-connected. Is the same true for infinitely many arcs? If not, what about a countable infinity of arcs?
 
Disjoint arcs, you mean?
 
12:59 PM
Pairwise disjoint arcs?
 
Ah, Balarka already asked
 
Also, it matters that it's "path-connected" and not "connected". I think that it's not too hard to show that, for countably many arcs, the complement must be connected.
 
My internet is being very slow today
I don't think that's true even for countably many arcs
 
I don't know how to prove the finitely many version for topologically embedded arcs though. For smooth arcs you can probably do it very easily because locally the arcs look like the x-axis in R^2 (immersion theorem).
Ah, I guess the same argument generalizes by Schoenflies (embedded arcs are ambient isotopic to a bit of the x-axis).
 
1:08 PM
Re my "it's not too hard to show" comment above: I take that back.
 
user84215
is it possible to have an uncountable set of real numbers that none of its element is a limit point respect to its elements ?
 
user84215
why ?
 
Isn't that the same as the question here: math.stackexchange.com/questions/310113/…
which Alessandro linked to above
 
user84215
no
 
Oh, wait, "limit point" is different from "accumulation point", isn't it
 
1:14 PM
is it...?
 
Actually, I have no idea
 
user84215
it is not important. I want none of its element is limit point respect to "its element" ?
 
In some contexts they're definitely the same...
Oh well
 
@aminliverpool With respect to the set's elements, you mean?
 
@aminliverpool What does that even mean
 
1:17 PM
In any case, you can't have an uncountable set made up of only isolated points.
 
Maybe it would be helpful if you gave your definition of a limit point and such
 
user84215
that is not respect to the bigger set that is real numbers
 
What does "respect to" mean???
 
user84215
a limit point of a set S in a topological space X is a point x (which is in X, but not necessarily in S) that can be "approximated" by points of S in the sense that every neighbourhood of x with respect to the topology on X also contains a point of S other than x itself.
 
So you want X to be the set S, rather than the set of real numbers?
 
user84215
1:20 PM
yes
 
That's the same as letting X be the set of real numbers and only looking for limit points in S.
 
user84215
no
 
Why
Do you know what a subspace topology is
 
@aminliverpool You're just looking at S with the subspace topology from X.
 
user84215
because maybe the sequence used to define limit points dont lie in S
 
1:23 PM
> can be "approximated" by points of S
 
user84215
ok
 
@aminliverpool Also, see where it says "also contains a point of S other than x itself"
@aminliverpool Note also that your definition doesn't explicitly mention sequences
But, nah, the sequence would have to lie in S.
 
(And that not all limit points of S are the limit of a sequence in S)
In a generic topological space I mean, sequences are fine in R
 
@AlessandroCodenotti In this case they're the same, though.
Yeah.
 
user84215
What is the conclusion ?
 
1:29 PM
@aminliverpool The only way for it to be false is if you had an uncountable set made of only isolated points. This can be shown to be impossible
so every uncountable set must have a limit point.
(It's impossible for an uncountable set to be made of only isolated points, because then you could define a surjective function from a subset of rationals to the set, contradicting its uncountability.
The function is defined by mapping every rational q to the largest element of the set less than or equal to the q, if it exists. (To be cont'd)
 
user84215
I want the elements are not limit point respect to set S itself.
 
@aminliverpool I know.
@aminliverpool But do you agree that the only way for it to happen is if every element is an isolated point (respect to S)?
 
user84215
yes
 
(Cont'd) This is surjective, because every point is isolated, meaning it has an open set around it containing no other elements of S. Thus, the rationals in the open set greater than that point get mapped to that point. QED.)
 
Hi there, I'm having some troubles coming up with a function that does the following: I have a range of values [0 to 4.000.000] and I need to split it up in 40 points. Linearly this would be simple each point would be a multiple of 4.000.000/40, Which is 0, 100.000, 200.000, ... However I need this to be somewhat more exponential, where the values start going up with higher numbers at the start, but at a lower pace at the end.
So it would become something like 0, 500.000, 900.000, 1.300.000 (this is just a random example :D)
 
user84215
1:44 PM
I have problem in your argument. next day I will continue the discusion
 
Hi guys
 
@DJanssens Maybe 4000000^(n/40)
 
I have a question that I would like to post here
on math.stackexchange
 
so that the 1st will be the fortieth root, the 20th will be the square root, and the last will be four million @DJanssens
 
but I have already asked it on ux.stackexchange
do you think that if I rearrange it would it be a good fit for this site?
2
Q: What is the 8.8 mm R Criterion Curved Design?

FormlessCloudCan you exlplain me what is the 8.8 mm R Criterion Curved Design? Apparently it is a criteria used by the UMi Touch phone. Why it uses 8.8 mm curves (what does this even mean? Do they mean that the radius of the fillets is 8.8 mm?) and what is the relation with the golden ratio? What is a cutting...

basically I would like to ask why using 8.8 mm ensure the golden ratio
and why using 45 degress ensures a cutting rate of 0.333% whatever that is
thanks
 
1:47 PM
A lot of the stuff about the golden ratio being "more pleasing to the eye" is BS
 
lol thanks
 
@AkivaWeinberger that's more or less what I needed, I think I need to turn it around though to 4000000^((40-n)/40) since I need the start to be bigger compared to the tail.
 
Oh, I see
 
so do you guys think that the company that designed that phone is just saying random things?
 
I dunno
I mean, they didn't make up the stuff about the golden ratio. It's been around for a long time. It's still BS, though
 
1:55 PM
@AkivaWeinberger That's indeed more or less what I needed, however it seems that the exponential function is a bit too powerfull, is there a way to influence the steepness of it :D
 
mmm okay thanks. I may try to ask a similar question on this site maybe
 
This is sort of a random question, but does anyone know any awesome math-related pick up lines (ie., if you're trying to get laid)? No, I'm actually serious.
2
 
focusing more on the mathematical aspect
 
@DJanssens
How about that? No exponential function, but you can vary the steepness by changing n.
(Click on the image to make it bigger)
The app is desmos.com, by the way
 
the calculator looks cool @AkivaWeinberger but it doesn't seem to plot it :/
 
2:05 PM
@DJanssens Change the scale? I put in 4000000 so it probably is out of the frame
2
 
@MathematicsStudent1122 You could probably get some inspiration by listening to "Finite simple group of order 2" by the Klein Four Group
 
@AkivaWeinberger that's pretty awesome! thanks for the site and tip
 
The $\int \frac{dx}{x}$ question today has had many views, and it got me wondering, is there an algebraic geometry motivated answer to it? I recall a section on Abelian integrals in Griffith's text...
 
What question
 
41
Q: The deep reason why integral of 1/x is a transcendental function (log)

John ZhengIn general, the indefinite integral of $x^n$ has power $n+1$. This is the standard power rule. Why does it "break" for $n=-1$? In other words, the derivative rule $$\frac{d}{dx} x^{n} = nx^{n-1}$$ fails to hold for $n=0$. Is there some deep reason for this discontinuity?

I'm just wondering if there's an algebraic geometry motivated explanation (which albeit would go over the OP's head)
(See for example, chapter 2 of Griffiths)
 
2:18 PM
@JamalS I wonder if you can connect it with the sinc function. $\sin(\pi x)/(\pi x)$ (extended to $0$ by continuity) is equal to $0$ for all integers except for $x=0$, where it equals $1$.
@MathematicsStudent1122 Something about laying tangent to someone's curves
3
 
Or blowing up a singularity on them...
 
2:52 PM
Are there open sets in $\Bbb R^2$ that are simply connected but not contractible?
 
@s.harp It actually has to be homeomorphic to R^2. This is the big conclusion of the uniformization theorem.
If you identify R^2 with C you moreover get that it has to be biholomorphic to a disk
 
does anyone have a good grasp in graph theory?
 
does it have to be the same as the disk or the same as the disk OR $\Bbb C$?
maybe im an idiot, but i thought they were not conformally equivalent
 
disk if your open set is proper. If you include all of C what you said is right, yes.
No, you're correct
 
gotcha
 
3:07 PM
Hi chat
 
Hello
 
What's up ? @ShaVulkia
 
@Astyx Analysis homework :P You?
 
Swimming pool in 20 minutes :)
 
Competitive swimming? :P
 
3:11 PM
No, thankfully not .. I'm bad at swimming
 
Haha, well that sounds good! Physical exercise is good :)
 
Just to do some sports in the week
 
I mean
the fact that you're going to do that for fun
:P
Yea I should do that too actually. But I have to choose between physical exercise or more sleep... And I've chosen sleep for now :P
 
Sleep is cool too :p
 
XD
quick question:
Let $(V,\rVert \lVert_V)$ be a $k$-dimensional linear space over $\mathbb R$. Choose a norm $\lVert \rVert$ for $\mathbb R^k$. Prove using Bolzano-Weierstrass that
$$
\inf_{\{\vec x\in\mathbb R^k:\lVert \vec x\rVert=1\}}\lVert\Phi(\vec x)\rVert_V>0.
$$
I've chosen the Euclidean norm on the $k$-dimensional Euclidean space $\mathbb R^k$. So we have
\begin{align}
\lVert\vec x\rVert=\sqrt{x_1^2+\dots+x_k^2}.
\end{align}
Consider a sequence $(\vec x_n)$ in $\mathbb R^k$ with $\Vert \vec x_n\Vert=1$ for each $n\in\mathbb N$. It's obvious that this sequence is bounden. By Bolzano-Weierstrass, we k
My teacher told me to use contradiction here
So I have to assume that
$\inf_{\{\vec x\in\mathbb R^k:\lVert \vec x\rVert=1\}}\lVert\Phi(\vec x)\rVert_V=0.$
 
3:17 PM
What's $\Phi$ ?
 
However, I don't know how to use this convergent subsequence.
oh right
it's a linear map:
$\Phi:\mathbb R^k\to V$
and it's a bijection
 
Oh right
If it wasn't a bijection that wouldn't be true
 
so basically the coordinate function :P
Or at least, the coordinate function could be an example
 
Well the linear map is continuous
So you have $||\Phi(x)||=0$, where $x$ is a limit point
 
Hi. A stupid question: I have $\displaystyle\lim_{x \to{+}\infty}{(x^4+7x+2)^c-x} = L$ where $0\neq L \in \mathbb{R}$. I have to compute $c$.
 
3:19 PM
wait, why is it continuous?
 
Because all linear maps are continuous in finite dimensionnal spaces
 
okay
 
How do I do compute c? I was thinking on simply define $c$ as $c = \dfrac{\log(x+k)}{\log(x^4 +7x+2)}$
 
That means $x=0$ because it's continuous
And from there I think you can find the contradiction :)
 
But how did you get $||\Phi(x)||=0$ ?
oh wait
how do you know that $\vec 0$ is the limit point?
because why does the subsequence have to converge to the infimum?
 
3:22 PM
It doesn't
Suppose you have a sequence such that $\Phi(x_n)$ goes to 0
 
Ohh
right
 
Then as you said it's closed and bounded so it has a limit point
(finite dimension, once again)
 
because if no sequence goes to o, then we couldn't have had the infimum equal to 0 in the first place
 
Yup
 
aha, okay
yea that helps!
thanks, I'm gonna write it out neatly now:)
 
3:23 PM
Glad to help
@Topologicalife What's $k$ ? Do you mean $L$ ?
 
Yeah.
 
Hello all, i have a question pertaining to behavior of 2nd order linear homogeneous ODE with constant coefficients. Please help if you can. Thanks :)

http://math.stackexchange.com/questions/2157986/behaviour-of-2nd-order-homogeneous-linear-ode-with-constant-coefficients
 
$x^{4} + 7x + 2 \sim x^4$ as $x\to \infty$
 
Yeah.
 
Or even $x^{4} + 7x + 2 = x^4 + O(x)$
 
3:27 PM
But yeah, and?
I don't get your point.
You can define $c = 1/4$ but then the limit is $0$.
 
Which is $x^4(1+O(x^{-3}))$
Then use Taylor expansions at 0
So you get $x^{4c}(1+O(x^{-3}))$
Well if the LHS converges then $c={1\over 4}$
Necessarily
 
Yeah. But then my limit is zero.
And remember:
11 mins ago, by Topologicalife
Hi. A stupid question: I have $\displaystyle\lim_{x \to{+}\infty}{(x^4+7x+2)^c-x} = L$ where $0\neq L \in \mathbb{R}$. I have to compute $c$.
 
So you have a contradiction
 
Indeed.
That's what is confusing me.
I think there doesn't exist $c$ such that.
 
Well that's what we just proved
 
3:31 PM
@BalarkaSen Here's another question: Must the complement of countably many compact line segments (not curved!), pairwise disjoint, be path-connected? :P
 
I gotta go now, I'll be back later
 
Have fun at the pool!
 
(I still have no idea what happens when we change "path-connected" to "connected")
 
Thanks @Astyx
Our ways are pretty the same.
 
I don't know where I gest stuck with.
I think that the full subcategory of torsion abelian groups is a Grothendieck abelian category.
 
3:37 PM
What happens if I choose $\{q\}\times [0,1]$ with $q\in\Bbb Q\cap(0,1)$ in $[0,1]^2$ and then cover the other squares in the plane in a similar fashion but turning 90 degrees between adjacent squares?
 
The generator is given by $\oplus_n\mathbb Z/n$.
 
4:05 PM
Let $(V,\rVert \lVert_V)$ be a $k$-dimensional linear space over $\mathbb R$. Let $\Phi:\mathbb R^k\to V$. Choose a norm $\lVert \rVert$ for $\mathbb R^k$. Prove using Bolzano-Weierstrass that
$$
\inf_{\{\vec x\in\mathbb R^k:\lVert \vec x\rVert=1\}}\lVert\Phi(\vec x)\rVert_V>0.
$$
I've chosen the Euclidean norm on the $k$-dimensional Euclidean space $\mathbb R^k$. So we have
\begin{align}
\lVert\vec x\rVert=\sqrt{x_1^2+\dots+x_k^2}.
\end{align}

I have one question. Assume $\inf_{\{\vec x\in\mathbb R^k:\lVert \vec x\rVert=1\}}\lVert\Phi(\vec x)\rVert_V=0$. Consider a sequence $(\vec x^{(n)}
 
@Alessandro That's a pretty good idea. In fact think about $[-1, 1] \times \{-1, 1\}$ with $\{-1-1/n\}_{n \in \Bbb N} \times [-1, 1]$ and $\{1+1/n\}_{n \in \Bbb N} \times [-1, 1]$. It looks like two line segments of equal length parallel to the x-axis and a bunch of lines accumulating to the line connecting their endpoints, at both ends.
Stuff in between $[-1, 1] \times \{-1\}$ and $[-1, 1] \times \{1\}$ should be disconnected from the world.
So literally take a square and replace a pair of two opposite sides by a bunch of lines accumulating to them, I mean
Everything is disjoint but if you try to get outside from the "interior" of the square you're gonna hit the lines which accumulate to the boundary sides
 
Never mind, I got it!
 
4:20 PM
@BalarkaSen @AlessandroCodenotti Nice!
 
Hello.. Is there anyone who can help me with geometry circle problem!?
 
-1
Q: Proving Partial Order

chickenwing44Testing .... Making sure this works. Tick tock

 
@MithleshUpadhyay I rolled back the edit
 
4:38 PM
@BalarkaSen neat! So even segments can beheave weirdly
 
Find five different positive integers such that the sum of any three terms divides the sum of the other two. Any hints?
 
Hello
Is anyone of you familiar with cryptography?
 
in this
angle POQ = 30
and angle PQO is 90
so shouldnt PQ be one third of PO instead of being half of it?
 
5:04 PM
@Evinda I'm
 
@Balarka if you're interested I found a reference for a simple construction of an uncountable subset of R containing no perfect set
 
@user8469759 I want to design an encryption system of type Feistel that has not the weakness that the pure message can be found, given the encypted message and also a pair of a pure message and its cryptogramm. The system should have the following parameters: size of block: 8 bits, length of key: 6 bits, number of rounds : 3.


The encyption algorithm computes the following:

$L_i=R_{i-1} \\ R_i=L_{i-1}+F(k_i, R_{i-1})$.

I have thought to pick $F(k,R)=AR+B+k$, where A is a 24x24 matrix, $k=(k_1,k_2,k_3), k_i \in \mathbb{F}_2^4$ and we would have to find specific A and B such that the orig
 
Hi again chat
 
what do you mean by "has not the weakness that pure message can be found"?
 
5:19 PM
@user8469759 I am looking at an exercise where it is the second question.

It is stated as follows:

Alice communicates with Bill using an encryption system of type Feistel. Specifically, each block has length 128 bits, the algorithm has 4 rounds and the common, secret key of Alice and Bill is $(k_1, k_2, k_3, k_4)$, where $k_i \in \mathbb{F}_2^{64}, i=1, \dots, 4$ is the key of the i-th round. The original message is divided into two parts, of 64 bits each , the left and the right , $L_0$ and $R_0$. Next, the encryption algorithm computes the following
 
@Astyx! I need your help XD
 
Do you now ? :p
 
Yes, it's still that same question! Let me just type it in math jax
@Astyx
I've proven that for some norm $\lVert\rVert_V$, there exist $c,C>0:$
$$
c\Vert \vec x\Vert\leq\Vert \Phi(\vec x)\Vert_V\leq C\Vert \vec x\Vert.
$$
Now I have to show that any two norms on $V$ are equivalent. My problem is that I've shown equivalence between two norms, that are not meant for the same vector space. I need something of the form:
$$
c\Vert \Phi (\vec x)\Vert_{V_2}\leq\Vert \Phi(\vec x)\Vert_{V_1}\leq C\Vert \vec x\Vert_{V_2}.
$$
How can I do that?
 
Let $N_1$ and $N_2$ be two norms on $V$
Then you just proved $c||\Phi^{-1}|| \le N_1 \le C||\Phi^{-1}||$
And $c||\Phi^{-1}|| \le N_2 \le C||\Phi^{-1}||$
 
Let me see what you did
I really don't get it. How are we going to use this inverse? I know that $\Phi$ is a continuous bijection, so both $\Phi$ and its inverse are strictly increasing or decreasing
but... I don't see it @Astyx
 
5:39 PM
What ?
A bijection isn't necessarilly increasing or decresing
As a matter of fact increase qnd decrease doesn't make sense in vector spaces
 
oh wait, that's only on $\mathbb R$
 
It is
Rewrite the inequalities I wrote so as to have $||\Phi^{-1}||$ in the center
 
The way I interpret you, that's impossible? @Astyx
I can only move one $|| \Phi^{-1}||$ to the middle
wait I'll show you
 
You have $c||\Phi{-1}|| \le N_1$ and $N_1\le C||\Phi{-1}||$
 
5:54 PM
And how can I combine $c$ and $C$? @Astyx
 
$\Bbb R$ is a field :)
 
oooooooohhhhhh
WAIT.
$\frac{N_2}{C}\leq \Vert\Phi^{-1}\Vert\leq\frac{N_1}{c}$ @Astyx
 
Right :)
Now you get the same for $N_2$ (different c and C though)
Using this you can compare $N_2$ and $N_1$
 
okay let me try!
Ahh, I got it. If we let $c_1, C_1$ belong to $N_1$, and $c_2,C_2$ to $N_2$, then we get
$
\frac{c_1}{C_2}N_2\leq N_1\leq\frac{C_1}{c_2}N_2
$
Thanks so much! @Astyx
 
Right
There another way to do this
 
6:03 PM
yea?
 
You can show that $||\Phi^{-1}||$ is a norm
And what you did is show that it's equivalent to any other norm
So all norms are equivalent
because equivalence between norms is (thankfully) an equivalence relation
 
but $\Vert \Phi^{-1}\Vert$ is a norm on $\mathbb R^k$, right?
 
No
It takes values in $V$, right ?
 
Does that mean that $V$ is the domain or the codomain?
because it's the inverse of $\Phi$, and we know that $\Phi:\mathbb R^k\to V$
 
Domain
 
6:07 PM
yea okay, but the domain doesn't matter?
 
Not sure what that means, English is not my native language :p
 
we take the norm of the assigned values by $\Phi^{-1}$
So $\Phi^{-1}(v)=\vec x$
so if we take the norm of $\Phi^{-1}(v)$, we take the norm of some $\vec x$
 
Yes
 
So it's a norm on $\mathbb R^k$, right
 
No, $||\cdot||$ is a norm on $\Bbb R^k$
$||\Phi^{-1}(\cdot)||$ takes vectors of $V$ and returns a positive real
So it's a norm on $V$
 
6:12 PM
Oh right, like that
 
Given a matrix $A=\begin{pmatrix}-3 & 5 \\ -2 & 2\end{pmatrix}$ and $X'=AX$, we can determine the direction portrait by looking at $\text{tr}(A)\pm\sqrt{\text{tr}(A)^2-4\det(A)}$
is there a way to draw the exact phase portrait with these information?
 
I'll be back, bye
 
Bye bye!
 
6:36 PM
@BalarkaSen @AlessandroCodenotti Easier construction (though unbounded): $\bigcup_{n=1}^\infty(\{1/n\}\times[-n,n])$. That is, a bunch of vertical line segments that approximate the line $x=0$.
Balarka's wins if we want it to be bounded, or if we want them all to have unit (or bounded) length
 
Let $X$ be a set and define $f : X → X$. Let $A ⊆ X$. Prove that $f^{-1}[A]=A⇔f(A)⊆A$ and $f^{-1}[A]=A⇔f^{-1}[A]⊆A$. How do I go about proving "<=" in both statements?
 
7:29 PM
@Akiva @Balarka there's this question somewhat related to our discussion, apparently removing countably many lines from $\Bbb R^3$ can't mess it up too badly, weird
 
7:40 PM
Hello, i have that $z$ is a limit point of $\phi_{t}(y)$ that is $z=\lim_{t\rightarrow+\infty} \phi_{t}(y)$ and we have that $f(\phi_{t}(y))$ is decreasing
can i deduce that $f(z)\leq f(y)$
 
7:56 PM
Does the harmonic series approach positive infinity?
 
8:07 PM
yes. try using the comparison test to convince yourself
 
8:22 PM
@JingWeng Yes. Details can be found here:
@Sirmimer That doesn't seem to make sense. Let $X=\Bbb R$, and $f(x)=x/2$. Then $f([0,1])=[0,\frac12]\subseteq[0,1]$, but $f^{-1}[[0,1]]=[0,2]\ne[0,1]$.
 
8:47 PM
i'm here
 
Hi Zach
New to this site and is still figuring out how to use it...
 
hi @Ran
you'll get the hang of it pretty fast :P
 
Well. Answered and asked a few questions. I feel the people here are exceptionally nicer than.....well, most of online forums I know
 
well, it's nice to hear you feel that way :P
 
Yep. Math makes people nice~
 
8:52 PM
That's due to the difference of functionality of MSE and online forums.
 
Agreed. But for example, I come from China and there is a site in China that serves similar purposes. Still people are very scary and always condemn the people for asking "simple and stupid" questions
By the way, I have a question to ask. I want to ask a question about reference for Mackey topology in general. Especially that when properties in weak topologies (such as compactness) implies that in Mackey topology. Is this too broad to ask as a question in MSE?
 
I think that it's okay, although I don't know what you're talking about.
with a tag reference-request
You needn't be too serious about their comments of simplicity and stupidity. It just indicates that they don't want to answer, or that the question itself is below the threshold of the forum. There is little meaning to say a question is stupid without reference to the questioner.
 
9:24 PM
Is it always true that \sum |r_n| = \sum r+_n -\ sumr-_n ?
where r+_n = |r_n|+r_n / 2 and r-_n = |r_n | -r_n /2
 
9:48 PM
hey @Akiva
 
im studying spectral theorem now
 
I don't think I know that one
 
it says that the eigenvectors of a symmetric linear map form an orthonormal basis for $\Bbb R^n$
 
Ah, right
 
9:50 PM
(and, thus, that the eigenvalues are all real)
 
I wonder why it's called that
"Spectral"
 
well it doesn't have to do with prime ideals :P
@AkivaWeinberger if i make it in but can't go this year, am i still an "alum"?
 
I don't think so
I don't know if that means you have to retake the quiz, though
 
:/
 
@ZachHauk You should email Marisa or someone like that
 
9:52 PM
that's what i mean
 
Someone who works there
 
Marisa replied to me about financial aid
she said it's very flexible, and we would only have to pay as much as we can afford
 
How does one prove that the isometry group is isomorphic to the group of symetries of the unit sphere ?
 
isometries of what metric space...
 
Finite dimensional vector space
 
9:52 PM
like, any metric?
or euclidean norm
 
Any I'd say
 
ah
 
It seems quite intuitive
 
What about translations
Do you want origin-persevering isometries?
 
Yes
 
9:54 PM
If you preserve the origin, then the unit sphere maps to itself
 
Yes
 
so it bijects to the group of symmetries of the sphere that way
 
Exactly, how do you show it's a bijection ?
 
Find an inverse
 
The injective property is quite straightforward
 
9:56 PM
If $f$ is a symmetry of the sphere, how to you get the symmetry $g$ of the space corresponding to it?
 
You use homogeneity of the norm I guess
 
Make $g(\alpha\hat u)=\alpha f(\hat u)$ or something
Extend $f$ linearly
 
Not really linearly ?
 
Oh, right
Extend it… multiplication-by-scalars-y?
 
Homogeneously
 
9:58 PM
That.
 
Or homogeneous-of-degree-1ly to be more precise
 
oh i taught a kid induction today
 
Now show that it's really an inverse, and that it's a group homomorphism
 
and i also figured out how bad of a teacher i am
 
@ZachHauk Cool
@ZachHauk Less cool
 
9:59 PM
Yeah that's the tricky part
 
i always flub words and don't know how to phrase things
 
Except for the group part
 

« first day (2395 days earlier)      last day (2627 days later) »